summaryrefslogtreecommitdiff
path: root/macros/latex/contrib/litesolution/doc/litesolution-demo.tex
blob: ce9319cace54a141d07b45e8a2ae71ad3afa198c (plain)
1
2
3
4
5
6
7
8
9
10
11
12
13
14
15
16
17
18
19
20
21
22
23
24
25
26
27
28
29
30
31
32
33
34
35
36
37
38
39
40
41
42
43
44
45
46
47
48
49
50
51
52
53
54
55
56
57
58
59
60
61
62
63
64
65
66
67
68
69
70
71
72
73
74
75
76
77
78
79
80
81
82
83
84
85
86
87
88
89
90
91
92
93
94
95
96
97
98
99
100
101
102
103
104
105
106
107
108
109
110
111
112
113
114
115
116
117
118
119
120
121
122
123
124
125
126
127
128
129
130
131
132
133
134
135
136
137
138
139
140
141
142
143
144
145
146
147
148
149
150
151
152
153
154
155
156
157
158
159
160
161
162
163
164
165
166
167
168
169
170
171
172
173
174
175
176
177
178
179
180
181
182
183
184
185
186
187
188
189
190
191
192
193
194
195
196
197
198
199
200
201
202
203
204
205
206
207
208
209
210
211
212
213
214
215
216
217
218
219
220
221
222
223
224
225
226
227
228
229
230
231
232
233
234
235
236
237
238
239
240
241
242
243
244
245
246
247
248
249
250
251
252
253
254
255
256
257
258
259
260
261
262
263
264
265
266
267
268
269
270
271
272
273
274
275
276
277
278
279
280
281
282
283
284
285
286
287
288
289
290
291
292
293
294
295
296
297
298
299
300
301
302
303
304
305
306
307
308
309
310
311
312
313
314
315
316
317
318
319
320
321
322
323
324
325
326
327
328
329
330
331
332
333
334
335
336
337
338
339
340
341
342
343
344
345
346
347
348
349
350
351
352
353
354
355
356
357
358
359
360
361
362
363
364
365
366
367
368
369
370
371
372
373
374
375
376
377
378
379
380
381
382
383
384
385
386
387
388
389
390
391
392
393
394
395
396
397
398
399
400
401
402
403
404
405
406
407
408
409
410
411
412
413
414
415
416
417
\chapter{A Sample for \pkg{LiteSolution} Template}
\fancyhead[L]{\color{H6}\kaishu\faIcon{atom}\;2023年\titlelogo{https://sci.hdu.edu.cn}{HDU}「大学物理2」期中模拟}
\fancyhead[R]{\color{H6}\kaishu\rightmark\,}

\date{2023年12月3日}{大学物理教学团队}{A0715012}
{\href{https://qm.qq.com/q/UPbGudx8cK}{\textbf{物理問題作}}}
{https://ctan.org/pkg/litesolution}{CTAN}
{https://github.com/xiamyphys/LiteSolution}{GitHub}

\section{选择题(每题3分,共36分)}
\begin{choice}{D}[弹簧振子]
    一劲度系数为$k$的轻弹簧,下端挂一质量为$m$的物体,系统的振动周期为$T_1$. 若将此弹簧截去一半的长度,下端挂一质量为$m/2$的物体,则系统振动周期$T_2$等于
\begin{tasks}(4)
    \task $2T_1$
    \task $T_1$
    \task $\frac{T_1}{\sqrt2}$
    \task $\frac{T_1}{2}$
\end{tasks}
\end{choice}
\begin{solution}*
    弹簧的劲度系数与长度成反比,所以剪断一半后劲度系数变为$2k$;根据弹簧振子的周期表达式$T=2\pi\sqrt{\frac mk}$可知此时的周期$T_2=2\pi\sqrt{\frac{m/2}{2k}}=\frac{T_1}{2}$.\sokka{D}
\end{solution}

\begin{choice}{B}[单摆]
    把单摆摆球从平衡位置向位移正方向拉开,使摆线与竖直方向成一微小角度$\theta$,然后由静止放手任其振动,从放手时开始计时.若用余弦函数表示其运动方程,则该单摆振动的初相为
    \begin{tasks}(4)
        \task $\theta$
        \task $0$
        \task $\frac\pi2$
        \task $-\pi$
    \end{tasks}
\end{choice}

\vskip-2ex
\begin{paracol}{2}
\begin{choice}{A}[平面简谐波的波函数]
    一平面简谐波,波速$u=5\mathrm{m/s}$,$t=3\mathrm{s}$时波形曲线如图,则$x=0$处质点的振动方程可能为
\begin{tasks}(2)
    \task $y=2\ee{-2}\cos\ab(\frac12\pi t-\frac12\pi)$
    \task $y=2\ee{-2}\cos\ab(\frac12\pi t+\frac12\pi)$
    \task $y=2\ee{-2}\cos(\pi t+\pi)$
    \task $y=2\ee{-2}\cos\ab(\pi t-\frac32\pi)$
\end{tasks}
\end{choice}
\switchcolumn\centering
\vfill
\begin{tikzpicture}
    \draw [->,thick] (-1.2,0) -- (4.8,0) node [anchor=south] {$x(\mathrm{m})$};
    \draw [->,thick] (0,-1.8) -- (0,1.8) node [anchor=west] {$y(\mathrm{m})$};
    \draw [very thick] (-.8,0) sin (0,-1) node [anchor=north west] {$-2\ee{-2}$} cos (.8,0) sin (1.6,1) cos (2.4,0) sin (3.2,-1) cos (4,0) sin (4.8,1);
    \draw [thick,->] (1.6,1.2) -- (2.4,1.2) node [midway,anchor=south] {$u$};
    \foreach \a in {5,10,...,25}
    \node [anchor=north,xshift=0.16*\a cm] at (0,0) {\a};
\end{tikzpicture}
\vfill
\end{paracol}
\vspace{-.75em}
\begin{solution}*
    由图可知$A=0.02\mathrm{m},\ \omega=\frac{2\pi u}{\lambda}=\frac12\pi$.由于原点处$v>0$,所以初相$\varphi=-\frac\pi2$.\sokka{A}
\end{solution}

\begin{choice}{A}[增透膜]
    一艘油船行经我国台湾岛东部海域时发生石油泄漏,在海面上形成大片油膜,太阳光在头顶正射时,救授人员乘直升飞机从上往下看,发现油膜对$552\mathrm{nm}$波长的可见光反射形成干涉相长而最亮,则可以推测该区域油膜厚度可能为多少?(设石油折射率$n=1.2$,海水折射率$n=1.3$)
\begin{tasks}(4)
    \task $460\mathrm{nm}$
    \task $552\mathrm{nm}$
    \task $345\mathrm{nm}$
    \task $425\mathrm{nm}$
\end{tasks}
\end{choice}
\begin{solution}
    \begin{itemize}
        \item 由于$n_{\text{空}}>n_{\text{海}}>n_{\text{油}}$,所以石油两个表面反射光光程差为$\delta=2ne$.
        \item 使反射光干涉相长,即$2ne=k\lambda$. A选项刚好满足$k=2$时,$e_{\min}=2\cdot\frac{\lambda}{2n}=460\mathrm{nm}$.
    \end{itemize}
\end{solution}

\begin{choice}{C}[光程和光程差]
    在相同的时间内,一束波长为$\lambda$的单色光在空气中和在玻璃中
    \begin{tasks}(2)
        \task 传播的路程相等,走过的光程相等
        \task 传播的路程相等,走过的光程不相等
        \task 传播的路程不相等,走过的光程相等
        \task 传播的路程不相等,走过的光程不相等
    \end{tasks}
\end{choice}
\begin{solution}*
    光程的定义:在相同时间内光线在真空中传播的距离.题目中光传播时间相同,故光程相等;又因为光在两种介质中的传播速度不同,所以在相同的时间内传播的路程不相等.\sokka{C}
\end{solution}

\begin{choice}{C}[多普勒效应]
    一观察者站在铁路旁,一火车以$30\mathrm{m/s}$的速度向他驶来并发出频率为$440\mathrm{Hz}$的汽笛声. 已知空气中声速为$330\mathrm{m/s}$,问观察者听到的火车频率为
\begin{tasks}(4)
    \task $403\mathrm{Hz}$
    \task $480\mathrm{Hz}$
    \task $484\mathrm{Hz}$
    \task $528\mathrm{Hz}$
\end{tasks}
\end{choice}
\begin{solution}
    已知多普勒效应观察者(Observer)和发射源(Source)的的频率关系为
    \[\nu=\frac{u\pm v_o}{u\mp v_s}\nu_0\]

    $v_o$为观察者速度,接近为$+$,远离为$-$;$v_s$为发射源速度,接近为$-$,远离为$+$.观察者静止,其所听频率为
    \[\nu=\frac{330}{330-30}\times 440\mathrm{Hz}=484\mathrm{Hz}\]

    \sokka{C}
\end{solution}

\vskip-2ex
\begin{paracol}{2}
\begin{choice}{C}[弗琅禾费衍射]
    在如图所示的单缝弗琅禾费衍射实验中,若将单缝沿透镜光轴方向向透镜平移,则屏幕上的衍射条纹
\begin{tasks}(5)
    \task 间距变大\!
    \task 间距变小\!
    \task 不变化
    \task* 间距不变,明暗纹交替
\end{tasks}
\end{choice}
\switchcolumn\centering
\vfill
\begin{tikzpicture}
    \filldraw [fill=gray] (0,1.5) rectangle (0.2,0.5);
    \filldraw [fill=gray] (0,-1.5) rectangle (0.2,-0.5);
    \draw [thick,densely dashed] (-1.2,0)--(3,0);
    \draw [thick] (3,1.5)--(3,-1.5);
    \filldraw [fill=white] (1.5,0) ellipse (0.1 and 1.2);
    \foreach \y in {0.5,-0.5}
    \draw [thick,->,yshift=\y cm] (-1,0)--(0,0);
    \length{(1.5,-1.4)}{(3,-1.4)}{$f$}{(0.25,0)}
    \node [anchor=south] at (-1.2,0.5) {$\lambda$};
    \node [anchor=south] at (1.5,1.2) {$L$};
    \node [anchor=west] at (3,1.5) {Screen};
\end{tikzpicture}
\vfill
\end{paracol}
\vspace{-.75em}
\begin{solution}*
    条纹间距只与波长、焦距、缝宽有关,入射光方向不变,所以条纹间距、位置不变. \sokka{C}
\end{solution}

\begin{choice}{A}[牛顿环]
    牛顿环干涉装置上平凸透镜在垂直于平板玻璃的方向上,逐渐向下平移(靠近玻璃板)时,反射光形成的干涉条纹的变化情况是
    \begin{tasks}(2)
        \task 环纹向边缘扩散,环数不变
        \task 环纹向边缘扩散,环数增加
        \task 环纹向中心靠拢,环数不变
        \task 环纹向中心靠拢,环数增加
    \end{tasks}
\end{choice}
\begin{solution}*
    对于某条环,其光程差是确定的,所以环数不变;向边缘扩散光程差增大,可抵消透镜下移时导致的光程差减小.\sokka{A}
\end{solution}

\begin{choice}{B}[最大分辨力]
    假设用FAST装置探测波长为$20\mathrm{cm}$的宇宙射电信号,FAST望远镜的镜面直径为$500\mathrm{m}$,则装置的最小分辨角为
\begin{tasks}(4)
    \task $9.76\ee{-4}$
    \task $4.88\ee{-4}$
    \task $2.44\ee{-4}$
    \task $4.00\ee{-4}$
\end{tasks}
\end{choice}
\begin{solution}*
    $\theta=\frac{1.22\lambda}{D}=4.88\ee{-4}\mathrm{rad}$.\sokka{B}
\end{solution}

\vskip-2ex
\begin{paracol}{2}
    \begin{choice}{B}[双缝干涉]
        在双缝干涉实验中,屏幕$E$上的$P$点是明纹.若将缝$S_2$盖住,并在$S_1S_2$连线的垂直平分面处放一高折射率反射面$M$,如图所示.则此时$P$点
    \begin{tasks}(2)
        \task $P$点仍为明条纹
        \task $P$点为暗条纹
        \task 不能确定$P$点是明纹还是暗纹
        \task 无干涉条纹
    \end{tasks}   
    \end{choice}
    \switchcolumn\centering
    \vfill
    \begin{tikzpicture}[decoration={markings,mark=between positions .2 and .8 step 18mm with {\arrow{stealth}}}]
        \draw [very thick] (0,1.5)--(0,-1.5);
        \draw [thick,postaction=decorate] (-1,0)--(0,0.5)--(2,0.8);
        \draw [thick] (0,0.5)--(0.8,0);
        \draw [thick,postaction=decorate] (-1,0)--(0,-0.5)--(2,0.8);
        \draw [very thick] (0,0)--(1.8,0);
        \draw [very thick] (2,1.8)--(2,-1.8);
        \fill [pattern=north west lines] (0,0) rectangle (1.8,-0.2);
        \fill [pattern=north east lines] (2,1.8) rectangle (2.2,-1.8);
        \node [anchor=east] at (-1,0) {$S$} node [anchor=south east] at (0,0.5) {$S_1$} node [anchor=north east] at (0,-0.5) {$S_2$};
        \node [anchor=north] at (0.8,-0.2) {$M$} node [anchor=west] at (2.2,0.8) {$P$} node [anchor=north] at (2.1,-1.8) {$E$};
        \node at (-1,0) {$\times$};
    \end{tikzpicture}
    \vfill
\end{paracol}
\vspace{-.75em}
\begin{solution}*
    $S_1MP$、$S_2MP$长度相等,但平面镜使在反射中一条光路发生半波损失,两条光路的相位差变化$\pi$,所以$P$点由原来的明纹变为暗纹.\sokka{B}
\end{solution}

\begin{choice}{A}[迈克尔逊干涉仪]
    如果使迈克尔逊干涉仪的动镜移动$0.233\mathrm{mm}$,观察到$792$个条纹的移动,则所用照明单色光源的波长是多少?
\begin{tasks}(4)
    \task $588\mathrm{nm}$
    \task $294\mathrm{nm}$
    \task $442\mathrm{nm}$
    \task $552\mathrm{nm}$
\end{tasks}
\end{choice}
\begin{solution}*
    移动带来的光程差满足$\delta=2d=N\lambda$,由此得$\lambda=\frac{2d}{N}=588.38\mathrm{mm}$.\sokka{A}
\end{solution}

\begin{choice}{B}[光栅]
    某元素的特征光谱中含有波长分别为$\lambda_1=450\mathrm{nm},\ \lambda_2=750\mathrm{nm}$的谱线,在光栅光谱中这两种波长的谱线有重合现象,重叠处$\lambda_2$的谱线级数将是
\begin{tasks}(4)
    \task $2,\ 4,\ 6,\ 8,\cdots$
    \task $3,\ 6,\ 9,\ 12,\cdots$
    \task $4,\ 8,\ 12,\ 16,\cdots$
    \task $5,\ 10,\ 15,\ 20,\cdots$
\end{tasks}
\end{choice}
\begin{solution}*
    由光栅方程$d\sin\theta=k_1\lambda_1=k_2\lambda_2$得$k_2=\frac{k_1\lambda_1}{\lambda_2}$,取整值得$k_2=3,6,9,12,\cdots$.
\end{solution}

\section{填空题(共18分)}
\begin{problem}[弹簧振子][3]
    当弹簧振子以频率$f$做简谐振动时,它的动能的变化频率为\ans{$2f$}.
\end{problem}
\begin{solution}*
    动能和势能变化趋势相反,所以二者变化频率相同. 势能$E_p\propto x^2$,由于$x$是周期为$T$的余弦函数,所以$x^2$的周期为$\frac T2$,即势能的变化频率等于动能的变化频率等于$2f$.
\end{solution}

\begin{problem}[驻波][3]
    在均匀介质中,一列余弦波沿$Ox$轴传播,波动方程为$y_1=A\cos\ab(2\pi t+\frac{2\pi x}3)$ (SI),在$x=1\mathrm{m}$处反射,反射点为固定端,则反射波和入射波产生的驻波表达式为\ans{$2A\cos{\ab(2\pi t+\frac{7\pi}6)}\cos{\ab(\frac{2\pi x}{3}-\frac{7\pi}{6})}$}.
\end{problem}
\begin{solution}
\begin{itemize}
    \item 考虑反射带来的半波损失,$x=1\mathrm{m}$处反射波的振动方程为$y_{10}=A\cos\ab(2\pi t+\frac{2\pi}{3}+\pi)$.
    \item 反射后传播方向改变,考虑以$x=1$处为参考点需坐标变换$x'=x-1$,所以反射波的表达式为
    \[y_2=A\cos\ab(2\pi t+\frac{2\pi}{3}-\frac{2\pi(x-1)}3+\pi)=A\cos\ab(2\pi t-\frac{2\pi x}{3}+\frac{7\pi}3)\]
    \item 驻波表达式$y=y_1+y_2=2A\cos{\ab(2\pi t+\frac{7\pi}6)}\cos{\ab(\frac{2\pi x}{3}-\frac{7\pi}{6})}$.
\end{itemize}
\end{solution}

\begin{problem}[双缝干涉][6]
    如图所示,在双缝干涉实验中,若把一厚度为$e$、折射率为$n$的薄云母片覆盖在$S_1$缝上,中央明条纹将向\ans{上}移动;覆盖云母片后,两束相干光至原中央明纹$O$处的光程差为\ans{$(n-1)e$}.
\end{problem}
\begin{solution}*
    覆盖云母片后,通过$S_1$的光路光程差变大,为抵消这一变化中央明纹需上移使通过$S_2$的光路变长;原光程差为零,现光程差即云母片带来的光程差$\delta=ne-e=(n-1)e$.
\end{solution}

\begin{problem}[牛顿环][3]
    若把牛顿环装置(都是用折射率为$1.52$的玻璃制成的)由空气搬入折射率为$1.33$的水中,则干涉条纹\ans{变密}(变疏/变密).
\end{problem}
\begin{solution}*
    放入水中后每条条纹的光程差变大,为抵消这一变化条纹需向中心收缩,所以干涉条纹变密.
\end{solution}

\begin{problem}[弗琅禾费衍射][3]
    在单缝夫琅禾费衍射实验中,波长为$\lambda$的单色光垂直入射在宽度为$a=6\lambda$的单缝上,对应于衍射角为$30^\circ$的方向,单缝处波阵面可分成的半波带数目为\ans{6}.
\end{problem}
\begin{solution}*
    由衍射公式$a\sin{\theta}=k\lambda$得$k=3$,可分成的半波带数目为$2k=6$.
\end{solution}

\section{计算题(共46分)}
\begin{paracol}{2}
\begin{problem}[平面简谐波的波函数][10]
    一列平面简谐波在媒质中以波速$u=5\mathrm{m/s}$沿$x$轴正向传播,原点$O$处质元的振动曲线如图所示.
\begin{enumerate}
    \item 求解$x=25\mathrm{m}$处质元的振动方程并画出该点振动曲线.
    \item 求解波动方程,并画出$t=3\mathrm{s}$时的波形曲线.
\end{enumerate}
\end{problem}
\switchcolumn\centering
    \vfill
    \begin{tikzpicture}
        \draw [->,thick] (-0.5,0)--(4,0) node [anchor=south] {$t(\mathrm{s})$} node [anchor=north] {\textcolor{H6}{$x(\mathrm{cm})$}};
        \draw [->,thick] (0,-1.5)--(0,1.5) node [anchor=west] {$y(\mathrm{cm})$};
        \draw[thick,H1] (0,0) sin (0.6,1) cos (1.2,0) sin (1.8,-1) cos (2.4,0) sin (3.0,1) cos (3.6,0);
        \draw [densely dashed] (0,1)--(.6,1) node [anchor=east,at start] {$2$};
        \node [anchor=north east] at (0,0) {$O$} node [anchor=south west] at (1.2,0) {$2$}  node [anchor=south east] at (2.4,0) {$4$};
        \draw [very thick,H2,dotted,line cap=round] (0,-1) cos (0.6,0) sin (1.2,1) cos (1.8,0) sin (2.4,-1) cos (3,0) sin (3.6,1);
        \draw [very thick,H6,densely dashed,line cap=round] (-.8,0) sin (0,-1) cos (.8,0) sin (1.6,1) cos (2.4,0) sin (3.2,-1) cos (4,0);
        \node [anchor=north] at (.8,0) {\textcolor{H6}{5}}  node [anchor=north] at (2.4,0) {\textcolor{H6}{15}};
    \end{tikzpicture}
    \vfill
\end{paracol}
\vspace{-.75em}
\begin{solution}
    \begin{enumerate}
        \item 由图知振幅$A=0.02\mathrm{m}$,角频率$\omega=\frac{2\pi}{T}=0.5\pi\mathrm{s}$. $t=0$时,$y=0,\ v>0$,初相$\varphi=-\frac\pi2$. 波动方程为\point{2}
        \[y=0.02\cos{\ab[\frac\pi2\ab(t-\frac{x}{5})-\frac\pi2]}\eqno\point{2}\]
        $x=25\mathrm{m}$处质元的振动方程为$y(x_0,t)=0.02\cos\ab(\frac\pi2t-\pi)$.\point{2}
        \item 波动方程见上问. $t=3\mathrm{s}$时的波形方程为
        \[y(x,t_0)=0.02\cos\ab(-\frac{\pi}{10}x+\pi)\eqno\point{2}\]
    \end{enumerate}
\end{solution}

\vskip-2ex
\begin{paracol}{2}
\begin{problem}[光程和光程差][8]\footnote{\kaishu 选自37th CPhO预赛试题第10题,原题未并提供参考图片.}
    一艘船(如图中$S$)在$25\mathrm{m}$高的桅杆($SS'$)上装有一天线(如图中$S''$),不断发射某种波长的无线电波,已知波长在$2-4\mathrm{m}$范围内,在高出海平面$150\mathrm{m}$的悬崖顶($OP$)上有一接收站$P$能收到这无线电波,但当那艘船驶至离悬崖底部$OS=2\mathrm{km}$时,接收站就收不到无线电波.设海平面完全反射这无线电波,求所用无线电波的波长.
\end{problem}
\switchcolumn\centering
\vfill
\begin{tikzpicture}[scale=0.8]
    \coordinate [label=below left:{$S$}] (S) at (0,0);
    \coordinate [label=above left:{$S'$}] (S') at (0,1);
    \coordinate [label=below left:{$S''$}] (S'') at (0,-1);
    \coordinate [label=below:{$M$}] (M) at (1.5,0);
    \coordinate [label=above:{$P$}] (P) at (6,3);
    \coordinate [label=below:{$O$}] (O) at (6,0);
    \draw [thick,line join=round,line cap=round] (P) -- (O) -- (S) -- (S');
    \draw [thick,dashed,line join=round] (P) -- (S) -- (S'') -- (M);
    \draw [very thick,line cap=round,densely dashed,H1] (S') -- (P);
    \draw [very thick,line cap=round,dotted,H1] (S') -- (M) --  (P);
    \length{(-.25,0)}{(-.25,1)}{$a$}{(0,0.25)}
    \pic ["$\theta$", draw, thick, angle radius=5mm, angle eccentricity=1.5] {angle=O--S--P};
    \pic ["$\theta'$", draw, thick, angle radius=5mm, angle eccentricity=1.5] {angle=O--M--P};
    \pic ["$\Delta\theta$", draw, thick, angle radius=10mm, angle eccentricity=1.5] {angle=S--P--M};
    \draw [thick] (S') -- (0.75,-0.5) coordinate [label=below:$K$] (foot);
    \pic ["$\theta$", draw, thick, angle radius=3mm, angle eccentricity=2] {angle=S''--S'--foot};
    \length{($(S'')+({0.5*sin(atan(2/3))},{-0.5*cos(atan(2/3))})$)}{($(foot)+({0.5*sin(atan(2/3))},{-0.5*cos(atan(2/3))})$)}{$\scriptstyle 2a\sin\theta$}{({0.5*cos(atan(2/3))},{0.5*sin(atan(2/3))})}
    \length{($(S)-(0,2)$)}{($(O)-(0,2)$)}{$2\mathrm{km}$}{(1.5,0)}
    \length{($(O)+(0.5,0)$)}{($(P)+(0.5,0)$)}{$150\mathrm{m}$}{(0,0.5)}
    \draw [dashed,H4] (S') --++ (6,0) -- (P);
    \draw [dashed,H6] (S'') --++ (6,0) -- (P);
\end{tikzpicture}
\vfill
\end{paracol}
\vspace{-.75em}
\begin{solution}
    考虑半波损失,经海平面反射的光波与直达$P$点的光波之间的光程差为
    \[\delta=2a\sin\theta+\frac\lambda2\eqno\point{3}\]
    由几何关系$\sin\theta=\sin\arctan\ab(\frac{150}{2000})=0.075$. 利用干涉相消条件得无线电波长为\point{2}
    \[\delta=\frac{2k+1}{2}\lambda,\ \lambda=\frac{2a\sin\theta}{k}\xlongequal[2<\lambda<4]{k=1}3.74\mathrm{m}\eqno\point{3}\]
    \textbf{本题中$\Delta\theta$不可忽略,不得认为$\theta=\angle PMO$! 否则会得到$k=2.18$.}
    \vskip1ex\hrule\vskip1ex
    另一种更精确的解法是直接计算两条光路的长度之差\footnote{值得一提的是,因上一种解法近似认为$S'K\bot S''K$,相对于这种解法有$7.73\ee{-3}\%$的误差(小数点第四位及以后不同).}
    \[\begin{aligned}
        \delta&=\overline{S''P}-\overline{S'P}+\frac\lambda2=\sqrt{\ab(\overline{SO})^2+\ab(\overline{OP}+\overline{SS''})^2}-\sqrt{\ab(\overline{SO})^2+\ab(\overline{OP}-\overline{SS''})^2}+\frac\lambda2\\
        &=\sqrt{2000^2+(150+25)^2}-\sqrt{2000^2+(150-25)^2}+\frac\lambda2=\frac{2k+1}{2}\lambda
    \end{aligned}\]

    解得$\lambda\xlongequal{k=1}3.74\mathrm{m}$.
\end{solution}

\begin{problem}[薄膜干涉][10]
波长为$\lambda=500\mathrm{nm}$的单色光垂直入射到\textbf{置于空气中}的上下表面平行的薄膜上,已知膜的折射率$n=1.25$,求反射光、透射光最强时膜的最小厚度.
\end{problem}
\begin{solution}*
    两个表面反射光光程差为$\delta=2ne+\frac\lambda2$. 分别由反射光干涉相长$\delta=k\lambda$和相消$\delta=\frac{2k+1}{2}\lambda$得\point{4}
    \[e_{\min_1}=\frac{\lambda}{4n}=100\mathrm{nm},\ e_{\min_2}=\frac{\lambda}{2n}=200\mathrm{nm}\eqno\point{4}\]
\begin{itemize}
    \item 反射光干涉相长时,反射光最强,膜的最小厚度为$e_{\min_1}=100\mathrm{nm}$.\point{1}
    \item 反射光干涉相消时,透射光最强,膜的最小厚度为$e_{\min_2}=200\mathrm{nm}$.\point{1}
\end{itemize}
\end{solution}

\vskip-2ex
\begin{paracol}{2}
\begin{problem}[光栅][12]
    如右图所示,$AB$之间的虚线为一透射式光栅,该光栅在$1\mathrm{mm}$内刻画有$500$条狭缝,单条狭缝的缝宽为$a=0.5\mu\mathrm{m}$,一波长为$\lambda=500\mathrm{nm}$的单色平行光斜入射在该光栅上,入射角$\theta=30^\circ$(从光栅光轴下方入射),在光栅后放置凸透镜和观察屏(屏位于透镜的焦平面处),问屏上能看到哪几级谱线?
\end{problem}
\switchcolumn\centering
\vfill
\begin{tikzpicture}[decoration={markings,mark=between positions .25 and .8 step 20mm with {\arrow{stealth}}}]
    \filldraw [thick,fill=gray] (0,1.6) rectangle (0.15,0.5);
    \draw [densely dashed,thick] (0,0.5)--(0,-0.5);
    \filldraw [thick,fill=gray] (0,-1.6) rectangle (0.15,-0.5);
    \draw [densely dashed,thick] (-1.5,0)--(2,0);
    \coordinate (b) at (-1.5,0);
    \coordinate (a) at (0,0);
    \coordinate (c) at (-1,-0.4);
    \coordinate (d) at (1.5,0.2);
    \coordinate (e) at (2,0);
    \foreach \a in {-0.5,-0.25,0,0.25,0.5}
    \draw [yshift=\a cm,thick,postaction=decorate,->] (-1,-0.4)--(0,0)--(1.5,0.2);
    \pic["$\theta$", draw=black, very thick, angle
    eccentricity=1.35, angle radius=24]{angle=b--a--c};
    \pic["$\varphi$", draw=black, very thick, angle
    eccentricity=1.35, angle radius=36]{angle=e--a--d};
    \node[anchor=east] at (0,0.6) {$A$} node[anchor=east] at (0,-0.6) {$B$};
\end{tikzpicture}
\vfill
\end{paracol}
\vspace{-.75em}
\begin{solution}*
    光栅常数$d=\frac{1\ee{-3}}{500}=2\mu\mathrm{m}$. 由于光栅方程\point{2}
    \[d(\sin\varphi-\sin\theta)=k\lambda\eqno\point{2}\]
\begin{itemize}
    \item 令$\varphi=\pm 90^\circ$得$k_{\min}=-6,\ k_{\max}=2$. 由缺级条件得$k'=\frac{d}{a}=\pm 4$,第$-4$级缺级.\point{4}
    \item 由于$\varphi$无法取到$\pm 90^\circ$,所以屏上可见主极大级次为$k=0,\ \pm 1,\ -2,\ -3,\ -5$.\point{4}
\end{itemize}
\end{solution}

\begin{problem}[驻波][6]
    由振动频率为$400\mathrm{Hz}$的音叉在两端固定拉紧的弦线上建立驻波.这个驻波共有三个波腹,其振幅为$0.30\mathrm{cm}$,波在弦上的速度为$320\mathrm{m/s}$.
\begin{enumerate}
    \item 求此弦线的长度.
    \item 若以弦的中点为坐标原点,试写出弦线上驻波的表达式.
\end{enumerate}
\end{problem}
\begin{solution}
\begin{enumerate}
    \item 由题意得弦长为$1.5$个波长,即$l=1.5\lambda=1.5\frac{u}{f}=1.2\mathrm{m}$.\point{2}
    \item 驻波的角频率$\omega=2\pi f=800\pi$,波矢$k=\frac{2\pi}{\lambda}=\frac{5}{2}\pi$. 设驻波的表达式为
    \[y=3\ee{-3}\cos\ab(800\pi t+\phi)\cos\ab(\frac52\pi x+\varphi)\eqno\point{2}\]
    中点$x=0$处是波腹,所以$\cos\varphi=1,\ \varphi=0\ \text{or}\ \pi$. 所以驻波的表达式为
    \[y=\pm 3\ee{-3}\cos\ab(800\pi t+\phi)\cos\ab(\frac52\pi x)\eqno\point{2}\]
    符号$\pm$对应$\varphi$的两个解,$\phi$由初始条件决定.
\end{enumerate}
\end{solution}